How do I derive this vector calculus identity?

Click For Summary
The discussion centers on deriving the vector calculus identity involving the curl of a vector field B. Key points include the acknowledgment that vector operators like ∇ cannot be treated as simple vectors, emphasizing the importance of maintaining the correct order of operations. Suffix notation is suggested as an efficient method for showing component equality between both sides of the equation. Participants reference useful vector calculus identities but note the lack of direct applicability to the problem at hand. The conversation concludes with a consensus on the need for careful component analysis in tackling the derivation.
Bright Liu
Messages
4
Reaction score
1
Homework Statement
As the title says, how can I get this equation?
Relevant Equations
tensor analysis
##(\nabla\times\vec B) \times \vec B=\nabla \cdot (\vec B\vec B -\frac 1 2B^2\mathcal I)-(\nabla \cdot \vec B)\vec B##
##\mathcal I## is the unit tensor
 
Physics news on Phys.org
You need to know these.
$$\mathbf a\times(\mathbf b\times\mathbf c)=\mathbf b(\mathbf a\cdot\mathbf c)-\mathbf c(\mathbf a\cdot\mathbf b)$$$$\mathbf a\cdot(\mathbf b\times\mathbf c)=\mathbf b\cdot(\mathbf c\times\mathbf a)=\mathbf c\cdot(\mathbf a\times\mathbf b)$$
 
  • Like
Likes etotheipi
Also note that since ##\mathbf{u} \times \mathbf{v} = -\mathbf{v}\times \mathbf{u}##, you have$$\begin{align*}(\mathbf{a} \times \mathbf{b}) \times \mathbf{c} &= - \mathbf{c} \times (\mathbf{a} \times \mathbf{b})\\ \\
&= \mathbf{a} (-\mathbf{c} \cdot \mathbf{b}) - \mathbf{b}(-\mathbf{c} \cdot \mathbf{a}) = -\mathbf{a}(\mathbf{c} \cdot \mathbf{b}) + \mathbf{b}(\mathbf{c} \cdot \mathbf{a})
\end{align*}$$
 
  • Like
Likes archaic
etotheipi said:
Also note that since ##\mathbf{u} \times \mathbf{v} = -\mathbf{v}\times \mathbf{u}##, you have$$\begin{align*}(\mathbf{a} \times \mathbf{b}) \times \mathbf{c} &= - \mathbf{c} \times (\mathbf{a} \times \mathbf{b})\\ \\
&= \mathbf{a} (-\mathbf{c} \cdot \mathbf{b}) - \mathbf{b}(-\mathbf{c} \cdot \mathbf{a}) = -\mathbf{a}(\mathbf{c} \cdot \mathbf{b}) + \mathbf{b}(\mathbf{c} \cdot \mathbf{a})
\end{align*}$$
Yes, but you cannot do that with a vector operator. ##\nabla\times\vec A## is not ##-\vec A\times\nabla##.
There’s a page of useful looking identities and their derivations at https://en.m.wikipedia.org/wiki/Vector_calculus_identities, but I do not see any that apply easily here.
 
  • Like
Likes etotheipi
Bright Liu said:
Homework Statement:: As the title says, how can I get this equation?
Relevant Equations:: tensor analysis

##(\nabla\times\vec B) \times \vec B=\nabla \cdot (\vec B\vec B -\frac 1 2B^2\mathcal I)-(\nabla \cdot \vec B)\vec B##
##\mathcal I## is the unit tensor

Suffix notation is generally the best method (this being a more efficient form of "show that each cartesian component of the left hand side is equal to the corresponding cartesian component of the right hand side").

In particular, <br /> \mathcal{I}_{ij} = \delta_{ij} = \begin{cases} 1 &amp; i = j \\ 0 &amp; i \neq j \end{cases} and
<br /> (\mathbf{a} \times \mathbf{b})_i = \epsilon_{ijk}a_jb_k where <br /> \epsilon_{ijk} = \begin{cases} 1 &amp; \mbox{$(i,j,k)$ is an even permutation of (1,2,3)} \\<br /> -1 &amp; \mbox{$(i,j,k)$ is an odd permutation of (1,2,3)} \\<br /> 0 &amp; \mbox{otherwise}\end{cases} together with the identity <br /> \epsilon_{ijk} \epsilon_{kpq} = \delta_{ip}\delta_{jq} - \delta_{iq}\delta_{jp}
 
  • Like
Likes etotheipi
haruspex said:
Yes, but you cannot do that with a vector operator. ##\nabla\times\vec A## is not ##-\vec A\times\nabla##.
There’s a page of useful looking identities and their derivations at https://en.m.wikipedia.org/wiki/Vector_calculus_identities, but I do not see any that apply easily here.

Interesting, you're right! My previous, naïve, view was that so long as you work with Cartesian components then it's generally safe to treat ##\nabla \equiv
\begin{pmatrix}
\frac{\partial}{\partial x} & \frac{\partial}{\partial y} & \frac{\partial}{\partial z}
\end{pmatrix}^T##, however you are certainly correct that it doesn't work here, considering that the order of the ##\frac{\partial}{\partial x_i}## and the ##A_j## definitely matters.

In that case, I'll step back and see how yourself and @pasmith approach the problem, for fear of misleading the OP. Working with the components as mentioned in #5 seems like a good idea!
 
  • Like
Likes archaic
Question: A clock's minute hand has length 4 and its hour hand has length 3. What is the distance between the tips at the moment when it is increasing most rapidly?(Putnam Exam Question) Answer: Making assumption that both the hands moves at constant angular velocities, the answer is ## \sqrt{7} .## But don't you think this assumption is somewhat doubtful and wrong?

Similar threads

  • · Replies 9 ·
Replies
9
Views
2K
Replies
2
Views
1K
  • · Replies 9 ·
Replies
9
Views
2K
  • · Replies 3 ·
Replies
3
Views
1K
  • · Replies 5 ·
Replies
5
Views
2K
  • · Replies 6 ·
Replies
6
Views
1K
  • · Replies 5 ·
Replies
5
Views
2K
  • · Replies 3 ·
Replies
3
Views
2K
  • · Replies 3 ·
Replies
3
Views
2K
  • · Replies 3 ·
Replies
3
Views
3K